0 Daumen
459 Aufrufe

Aufgabe:

Berechnen Sie die Volumina der folgenden Mengen:
(a) \( \left\{(x, y, z) \in \mathbb{R}^{3}: x, y, z \geq 0, x^{2}+y^{2} \leq 1, x+y+z \leq \sqrt{2}\right\} \),
(b) \( \left\{(x, y, z) \in \mathbb{R}^{3}: x^{2}+y^{2} \leq 1,0 \leq z \leq x^{2}+y^{2}\right\} \).
[Es gilt: \( \int \limits_{0}^{1} x \sqrt{1-x^{2}} d x=1 / 3 \).]

Problem:

Heeyy Mathefreaks. Ich habe ein wenig Probleme mit der oben stehenden Aufgabe. Ich weiß nicht, wie ich über die Mengen integrieren soll bzw., wie ich die Intervallgrenzen setzen soll. Mich würde auch interessieren, wie man generell an solche Aufgaben geht und dort die Intervallgrenzen taktisch klug setzt, sowie die Reihenfolge, in welcher man nach dx dy dz integriert. Wäre sehr nett, wenn da jemand paar Tipps hat. :)

Ansatz:
a) Es ist nach dem Volumina gefragt. Demendsprechend kann ich ein Dreifachintegral über 1 nutzen. Die unteren Intervallgrenzen für x,y,z sind 0.
b) Ja z ist größer 0 dieses mal :D Nach dem herausfinden des Mehrfachintegrals und vor dem Berechnen des Voluminas, macht es hier vermutlich sinn nach Polarkoordinaten umzustellen, oder ?

Avatar von

Ich weiß nicht, wie ich über die Mengen integrieren soll bzw., wie ich die Intervallgrenzen setzen soll

Wenn dir das in einer Prüfungssituation (z.B. Klausur) passiert, dann kannst du versuchen, einen anderen Lösungsweg zu beschreiten.
Hier : (a) ist ein halber Zylinder mit Höhe 2√2 , (b) ergibt sich aus einem Rotationskörper zur Funktion f(r)=r^2

1 Antwort

+1 Daumen
 
Beste Antwort

Als Beispiel zu a): x läuft von 0 bis 1 (letzteres sieht man aus der 2. Bed.). Schon hat man die Grenzen für das dx-Integral (das äußere). y läuft dann von 0 bis .... (2. Bed. umstellen), gibt dy-Integral. z läuft von 0 bis ... (3. Bed. umstellen), gibt dz-Integral (das innerste). Hinschreiben, ausrechnen.

Wenn man mit y anfängt (einfach) oder mit z (komplizierter), kommt man auf andere Integrale (ausprobieren um es zu lernen), aber alle Integrale haben natürlich denselben Wert.

Danach sollte es mit b) auch keine Probleme geben.

Avatar von 6,1 k

Danke das hat mir weitergeholfen.
Ich habe es sogar noch bisschen weiter durchdacht und sollte jetzt die reellen Grenzen gefunden haben. Stimmen Sie meiner Begründung (siehe Bild rechts) da zu ? Würde mich freuen, wenn Sie einen Blick drauf werfen könnten. :)PNG image.png

Nein, nur umstellen und als Grenze verwenden, nichts weiter überlegen. Also, obere Grenze für y ist \(\sqrt{1-x^2}\), fertig. Die Grenzen der inneren Integrale hängen oft von den Integrationsvariablen der äußeren Integrale ab. Daher Sorgfalt beim Integrieren: genau auf die Variablen achten.

"Die Grenzen der inneren Integrale hängen oft von den Integrationsvariablen der äußeren Integrale ab." - deswegen dachte ich, ich kann auf die y Grenzen auch schließen. Denn wenn x von 0 bis 1 geht und \(y von 0 bis \( \sqrt{1-x2} \), dann kann ich doch eigentlich die x-Grenze in \( \sqrt{1-x2} \) einsetzen, die den Term maximiert, für die oberere Grenze. Ich verstehe nicht wieso das nicht gehen sollte. Das Integral macht doch eigentlich nichts anderes sonst. Verstehen Sie mein Gedanken ?

Ich habe es auch so, wie Sie es sagten versucht. :)
Die Aufgabe sollte ohne Taschenrechner gelöst werden können. Allerdings kommt da 0,44...... raus. Und man muss ganz schlimme integrale lösen. Ich habe das mal hier unschön dargelegt. Werfen sie da noch bitte ein Blick drauf ? :) Kann man das auch einfach lösen ? Obwohl der Hinweis in der Aufgabe ja wichtig ist schon mal...

Text erkannt:

\( \begin{aligned} & \int \limits_{0}^{1} \int \limits_{0}^{\sqrt{1-x^{2}}} \int \limits_{0}^{\sqrt{2}-x-y} 1 d z d y d x \\ = & \int \limits_{0}^{1} \int \limits_{0}^{1-x^{2}} \sqrt{2}-x-y d y d x \\ = & \int \limits_{0}^{1} \sqrt{2} y-x y-\left.\frac{1}{2} y^{2}\right|_{0} ^{\sqrt{1-x^{2}}} d x \\ = & \int \limits_{0}^{1} \sqrt{2} \sqrt{1-x^{2}}-x \sqrt{1-x^{2}}-\frac{1}{2}\left(1-x^{2}\right) d x\end{aligned} \)

Achso und eine Frage noch. Damit ich das nicht falsch habe. Dass x von 0 bis 1 geht sieht man daran dass bei |x|≤\( \sqrt{1-y^2} \) unter der Wurzel keine negative Zahl stehen darf und y von unten mit 0 begrenzt ist, oder ?

Letzte Frage: ja.

Vorher: die Kopplung muss erhalten bleiben, x und y beide =1 geht nicht, ist nicht im Integrationsbereich.

Deine Rechnung stimmt nun, bis auf Flüchtigkeitsfehler. Das einzig etwas unangenehme Integral ist das erste (Tipp: substituiere x=sin(u)). Für das zweite gibt es ja den Tipp. Und Ergebnis ohne TR angeben.

PNG image.png

Text erkannt:


stimmt das für die b) Ich weiss nicht, wie ich bei der Transformation die Intervallgrenzen anpasst.

(b)
\( \text { b) } \begin{aligned} \left\{(x, y, z) \in \mathbb{R}^{3}:\right. & \left.x^{2}+y^{2} \leq 1,0 \leq z \leq x^{2}+y^{2}\right\} \\ \downarrow & \\ & \left(y \mid \leq \sqrt{1-x^{2}} \leadsto-\sqrt{1-x^{2}} \leq y \leq \sqrt{1-x^{2}}\right. \\ &  \quad-1 \leq x \leq 1 \end{aligned} \)
\( \begin{array}{l} \int \limits_{-1}^{1} \int \limits_{-\sqrt{1-x}}^{\sqrt{1-x^{2}}} \int \limits_{0}^{x^{2}+y^{2}} 1 d z d y d x \\ =\int \limits_{-1}^{1} \int \limits_{-\sqrt{1-x^{2}}}^{\sqrt{1-x^{2}}} x^{2}+y^{2} d y d x \end{array} \)
\( \begin{array}{ll} \text { Polarkoordinatentransformation: } & x=r \cos (\varphi) \\ & y=r \sin (\varphi) \end{array} \)
\( \begin{array}{l} =\int \limits_{0}^{1} \int \limits_{0}^{2 \pi}\left(r^{2} \cos ^{2}(\varphi)+r^{2} \sin ^{2}(\varphi)\right) d \varphi d r \quad \begin{array}{r} r=[0,1] \\ \varphi=[0,2 \pi] \end{array} \\ =\int \limits_{0}^{1} \int \limits_{0}^{2 \pi} 1 d \varphi d r=2 \pi \end{array} \)

Es hilft immer, das Gebiet zu skizzieren um zu sehen, wie die Variablen laufen.

Die Umstellung auf Polarkoordinaten hast Du richtig gemacht (Punktabzug für Unsinn wie \(r=[0,1]\)). \(x²+y²\) schreibt man natürlich gleich als \(r²\), wozu erst noch mit sin und cos umschreiben? Das ist eine Einladung für Fehler, den Du auch prompt gemacht hast. Also, schau nochmal genau hin. Ergebnis ist am Ende \(\frac\pi2\).

Da das Ergebnis jetzt dasteht : Mein Hinweis zu (b) von oben wäre wie folgt auszführen :

V = VZylinder - VRotationsparaboloid =  πr^2*h - π·z=01r2dz = π1^2*1 - π·z=01zdz = π-π/2  = π/2 .

Ahh Danke. Sie haben mir wirklich sehr geholfen. Ich glaube ich habe meinen Fehler erkannt. Da ja r=1 ist, habe ich einfach angenommen, ich kann das für r vor Vollendung der Integralrechnung einsetzen. Was ich nicht beachten habe ist, dass obwohl r fest = 1 ist, ich trotzdem einmal über r integriere. Bei mir kommt jetzt auch \( \frac{Π}{4} \) raus.
So zu rechnen ist aber auch wirklich nur möglich, wenn man Kugeln oder Kreise... in der Menge hat.

r ist nicht 1!!!! r ist die Integrationsvariable! Bloß weil in der Aufgabe was kreisförmiges mit Radius 1 drin vorkommt, muss nicht jedes r gleich 1 sein. Außerdem ist das Endergebnis \(\frac\pi2\), nicht \(\frac\pi4\).

Wie bei jedem bestimmten Integral steht im Endergebnis natürlich die Integrationsvariable nicht mehr drin, weil sie beim Einsetzen der Grenzen des Integrals (was r betrifft: hier 0 und 1) rausfällt. Und nicht, weil man da einen festen Radius aus der Aufgabenstellung einsetzt.

Da hast Du irgendwas grundlegend falsch verstanden. Schau, dass Du das unbedingt klärst, sonst wird es bei solchen Aufgaben nochmal Pannen geben.

Und ja, Polarkoordinaten sind hilfreich, wenn es um kreis- oder kugelförmige Objekte geht.

Ja ups, habe mich verschrieben habe \( \frac{π}{2} \) raus.
Alles klar, jetzt habe ich es. Mich hat zuerst verwirrt, dass Sie bemängelt haben, dass ich r=[0,1] geschrieben habe. Aber natürlich Sie haben vollkommen recht. Da muss element von. Sehr dummer Fehler. Weiß nicht, wie das passieren konnte haha

Hier noch mal meine Lösung für b und in a habe ich das versucht jetzt parallel zu machen.PNG image.png

PNG image 2.png

Du hast einen Fehler bei a) in der 2. Zeile gemacht. Wenn Deine 2. Zeile richtig wäre, dann hättest Du beim Integriere einen weiteren Fehler gemacht.

Abgesehen davon hast Du wieder, in beiden Fällen, denselben Fehler mit Deinem vermeintlichen r=1 gemacht. Lies nochmal meinen vorigen Kommentar dazu.

Ein anderes Problem?

Stell deine Frage

Willkommen bei der Mathelounge! Stell deine Frage einfach und kostenlos

x
Made by a lovely community